Mathcenter Forum  

Go Back   Mathcenter Forum > คณิตศาสตร์โอลิมปิก และอุดมศึกษา > คณิตศาสตร์อุดมศึกษา
สมัครสมาชิก คู่มือการใช้ รายชื่อสมาชิก ปฏิทิน ค้นหา ข้อความวันนี้ ทำเครื่องหมายอ่านทุกห้องแล้ว

ตั้งหัวข้อใหม่ Reply
 
เครื่องมือของหัวข้อ ค้นหาในหัวข้อนี้
  #31  
Old 19 พฤศจิกายน 2006, 01:58
warut warut ไม่อยู่ในระบบ
กระบี่ไร้สภาพ
 
วันที่สมัครสมาชิก: 24 พฤศจิกายน 2001
ข้อความ: 1,627
warut is on a distinguished road
Smile

วิธีทำที่คุณ passer-by แสดงให้ดูนั่นคือเฉลยของอาจารย์ใช่ไหมครับ สูตรใน (2) อันนั้นน่าสนใจมาก ตอนคิดข้อนี้ ผมไม่ได้สังเกตเห็นความสัมพันธ์อันนี้เลย ขอบคุณสำหรับโจทย์ดีๆ และข้อมูลเพิ่มเติมครับ
ตอบพร้อมอ้างอิงข้อความนี้
  #32  
Old 19 พฤศจิกายน 2006, 20:13
passer-by passer-by ไม่อยู่ในระบบ
ผู้พิทักษ์กฎทั่วไป
 
วันที่สมัครสมาชิก: 11 เมษายน 2005
ข้อความ: 1,442
passer-by is on a distinguished road
Post

ที่ผมทำให้ดูเป็น วิธีทำ(แบบย่อ)ที่ทำต่อจาก Hint ที่อาจารย์ให้ไว้อีกทีครับ ส่วนเฉลย น่าจะมาอาทิตย์ที่จะถึงนี่แหละครับ

โจทย์ข้อ12 นี้ ผมหั่นมาแค่บางส่วน (โจทย์แบบ full version เขาก็ถามตัว general formula ที่คุณ Warut ทำเหมือนกันครับ)

ว่าแล้วก็ต่อ ข้อ 13 ซึ่งผมมั่นใจว่า ผมไม่ได้ตั้งผิดกระทู้แน่นอน

Evaluate $$ \int_0^1 \big \{\frac{1}{x} \big \} \big \{\frac{1}{1-x} \big \} \,\, dx $$

Note : {a} แทน fractional part of a เช่น {5.187}= 0.187

__________________
เกษียณตัวเอง ปลายมิถุนายน 2557 แต่จะกลับมาเป็นครั้งคราว
ตอบพร้อมอ้างอิงข้อความนี้
  #33  
Old 20 พฤศจิกายน 2006, 15:42
warut warut ไม่อยู่ในระบบ
กระบี่ไร้สภาพ
 
วันที่สมัครสมาชิก: 24 พฤศจิกายน 2001
ข้อความ: 1,627
warut is on a distinguished road
Smile

โจทย์ข้อ 13. สวยมากครับ สวยจนผมอดใจไม่อยู่จริงๆ

เนื่องจาก $$ \int_0^{\frac12} \{ \frac1x \}\{ \frac{1}{1-x} \} \, dx = \int_{\frac12}^1 \{ \frac{1}{1-y} \}\{ \frac1y \} \, dy \quad ,y=1-x $$ ดังนั้น $$ \int_0^1 \{ \frac1x \}\{ \frac{1}{1-x} \} \, dx = 2 \int_0^{ \frac12 } \{ \frac{1}{x} \}\{ \frac{1}{1-x} \} \, dx $$ $$ =2 \int_1^{\infty} \{u+1\}\{ \frac1u +1\} \frac{du}{(u+1)^2} \quad , u= \frac1x -1 $$ $$ =2 \int_1^{\infty} \{u\}\{ \frac1u \} \frac{du}{(u+1)^2} $$ $$= 2 \int_1^{\infty} \frac{u- \lfloor u \rfloor }{ u(u+1)^2 } \, du $$ $$ =2 \lim_{n \to \infty} \sum_{k=1}^n \left( \int_k^{k+1} \frac{u-k}{ u(u+1)^2 } \, du \right) $$ $$ =2 \lim_{n \to \infty} \sum_{k=1}^n \left( \frac{1}{k+2} + k\ln k + k\ln(k+2) - 2k\ln(k+1) \right) $$ $$ =2 \lim_{n \to \infty} \left( \left( \frac13 + \frac14 + \cdots + \frac{1}{n+2} - \ln(n+1) \right) + n\ln \left( \frac{n+2}{n+1} \right) \right) $$ $$ =2 \left( \left( \gamma -1 -\frac12 \right) +1 \right) = 2\gamma -1 $$
ตอบพร้อมอ้างอิงข้อความนี้
  #34  
Old 20 พฤศจิกายน 2006, 21:12
passer-by passer-by ไม่อยู่ในระบบ
ผู้พิทักษ์กฎทั่วไป
 
วันที่สมัครสมาชิก: 11 เมษายน 2005
ข้อความ: 1,442
passer-by is on a distinguished road
Post

ข้อ 13 ถูกแล้วครับ
Alternative Solution

$$ \int_0^{\frac{1}{2}}\big \{ \frac{1}{x} \big\}\big \{ \frac{1}{1-x} \big\} \,\, dx= \sum_{n=2}^{\infty} \int_{\frac{1}{n+1}}^{\frac{1}{n}}\big \{\frac{1}{x} \big \} \big \{\frac{1}{1-x} \big \} \,\, dx = \sum_{n=2}^{\infty} a_n $$

And using the fact that , for each $ a_n $,
$ \quad \big \{ \frac{1}{x} \big \} =\frac{1}{x}-n \quad $ and $ \,\, \big \{ \frac{1}{1-x} \big\} = \frac{x}{1-x} $

Next, we'll compute partial sum and the result is

$$ s_N = \sum_{n=2}^N \big ( \frac{1}{n+1} + \ln \big ( ( 1+\frac{1}{n})^{n-1}\cdot (1-\frac{1}{n})^{n-1}\big) \big ) $$

After simplifying, we have $$s_N= \sum_{n=3}^N \frac{1}{n} - \ln N + \ln ( 1+\frac{1}{N})^{N} - \ln(1+\frac{1}{N})+\frac{1}{N+1} $$

Let $ N \rightarrow {\infty} $ and multiply with 2, we have got the answer

ต่อด้วยข้อ 14

ให้ n เป็นจำนวนนับ และ $ a_n , b_n $ เป็นลำดับของจำนวนจริงบวก ซึ่ง

$ a_{n+1} =a_n +\frac{1}{b_n}$ และ $ b_{n+1} =b_n +\frac{1}{a_n}$

หา n มาอย่างน้อย 1 ค่า ที่ไม่ใช่ 1 และทำให้ $ a_n + b_n > 1000 $
__________________
เกษียณตัวเอง ปลายมิถุนายน 2557 แต่จะกลับมาเป็นครั้งคราว

20 พฤศจิกายน 2006 21:18 : ข้อความนี้ถูกแก้ไขแล้ว 2 ครั้ง, ครั้งล่าสุดโดยคุณ passer-by
ตอบพร้อมอ้างอิงข้อความนี้
  #35  
Old 22 พฤศจิกายน 2006, 03:11
warut warut ไม่อยู่ในระบบ
กระบี่ไร้สภาพ
 
วันที่สมัครสมาชิก: 24 พฤศจิกายน 2001
ข้อความ: 1,627
warut is on a distinguished road
Smile

14. Note that $a_n, b_n >0$ for all $n\in \mathbb N$.

Without any loss of generality, we will assume from now on that $a_1\ge b_1$.

By induction, we can show that $a_n\ge b_n$ for all $n\in \mathbb N$.

Therefore, $a_{n+1}=a_n+\frac{1}{b_n}\ge a_n+\frac{1}{a_n}$ for all $n\in \mathbb N$.

In particular, $a_2\ge 2$ because the minimum of $x+\frac1x$ for positive real $x$ is 2.

Let $c_n:=a_n^2$. Hence, $$ c_{n+1} = a_{n+1}^2 \ge \left( a_n+ \frac{1}{a_n} \right)^2 = c_n + \frac{1}{c_n} +2 > c_n+2. $$ Since $c_2 = a_2^2 \ge 4$, we can prove by induction that $c_n>2n$ for $n\ge3$.

If $n\ge 1000^2/2 =500000$, then $c_n>1000^2$, and we have $$ a_n+b_n >a_n= \sqrt{c_n}>1000 $$ as required.
ตอบพร้อมอ้างอิงข้อความนี้
  #36  
Old 22 พฤศจิกายน 2006, 13:03
passer-by passer-by ไม่อยู่ในระบบ
ผู้พิทักษ์กฎทั่วไป
 
วันที่สมัครสมาชิก: 11 เมษายน 2005
ข้อความ: 1,442
passer-by is on a distinguished road
Thumbs up

เท่าที่สแกนคร่าวๆ ก็ไม่น่าจะมีปัญหาอะไรครับ เดี๋ยวว่างๆผมจะมาแปะวิธีทำของผมให้ดู ระหว่างนี้ใครได้ค่า n ที่น้อยกว่าวิธีของคุณ Warut ก็ตอบกันตามสบายเลยนะครับ
__________________
เกษียณตัวเอง ปลายมิถุนายน 2557 แต่จะกลับมาเป็นครั้งคราว
ตอบพร้อมอ้างอิงข้อความนี้
  #37  
Old 22 พฤศจิกายน 2006, 22:17
Timestopper_STG's Avatar
Timestopper_STG Timestopper_STG ไม่อยู่ในระบบ
ลมปราณคุ้มครองร่าง
 
วันที่สมัครสมาชิก: 22 มกราคม 2006
ข้อความ: 256
Timestopper_STG is on a distinguished road
Send a message via MSN to Timestopper_STG
Icon16

1)With an easily proof we'll find out that the centroid of any triangle
is the same point with it's medial triangle's centroid.
So now, We'll let $a_0,b_0,c_0$ be the midpoints of $a,b,c$ respectively
and also let $a_{n+1},b_{n+1},c_{n+1}$ be the midpoints of opposite side of $a_n,b_n,c_n$
$\displaystyle{\because a_{n+1}=\frac{b_n+c_n}{2},b_{n+1}=\frac{c_n+a_n}{2},c_{n+1}=\frac{a_n+b_n}{2}}$
$\rightarrow a_{n+1}+b_{n+1}+c_{n+1}=a_n+b_n+c_n,A+B+C=a_n+b_n+c_n$
If we continue drawing a medial triangle again and again we'll get $a_n=b_n=c_n=&centroid
$\therefore$coordinate of the centroid is $\displaystyle{\lim_{n\rightarrow\infty}} \frac{a_n+b_n+c_n}{3} = \frac{A+B+C}{3} = \left(\left(\frac{x_0+x_1+x_2}{3}\right), \left(\frac{y_0+y_1+y_2}{3}\right)\right)$
__________________
$$\int_{0}^{\frac{\pi}{2}}\frac{a\cos x-b\sin x}{a\sin x+b\cos x}dx=\ln\left(\frac{a}{b}\right)$$
BUT
$$\int_{0}^{\frac{\pi}{2}}\frac{a\cos x+b\sin x}{a\sin x+b\cos x}dx=\frac{\pi ab}{a^{2}+b^{2}}+\frac{a^{2}-b^{2}}{a^{2}+b^{2}}\ln\left(\frac{a}{b}\right)$$

22 ตุลาคม 2007 21:35 : ข้อความนี้ถูกแก้ไขแล้ว 1 ครั้ง, ครั้งล่าสุดโดยคุณ TOP
เหตุผล: เว้นวรรค LaTeX ที่เขียนติดกันยาวเกินไป
ตอบพร้อมอ้างอิงข้อความนี้
  #38  
Old 23 พฤศจิกายน 2006, 00:09
warut warut ไม่อยู่ในระบบ
กระบี่ไร้สภาพ
 
วันที่สมัครสมาชิก: 24 พฤศจิกายน 2001
ข้อความ: 1,627
warut is on a distinguished road
Smile

สำหรับข้อ 14. ถ้าอยากได้ $n$ น้อยกว่านี้ ผมยังลดได้อีกประมาณ 4 เท่าครับ แต่คราวนี้ต้องวิเคราะห์ $b_n$ ร่วมด้วย ถ้ามีเวลาจะแปะให้ดูครับ
ตอบพร้อมอ้างอิงข้อความนี้
  #39  
Old 23 พฤศจิกายน 2006, 22:26
Timestopper_STG's Avatar
Timestopper_STG Timestopper_STG ไม่อยู่ในระบบ
ลมปราณคุ้มครองร่าง
 
วันที่สมัครสมาชิก: 22 มกราคม 2006
ข้อความ: 256
Timestopper_STG is on a distinguished road
Send a message via MSN to Timestopper_STG
Post

\[
\begin{array}{l}
15)Let\;a_{n + 1} \;be\left\{ \begin{array}{l}
a_n + \frac{1}{n},a_n^2 < 2 \\
a_n - \frac{1}{n},a_n^2 > 2 \\
\end{array} \right.and\;also\;given\;a_1 = 1 \\
Show\;that\left| {a_n - \sqrt 2 } \right| < \frac{1}{n},\forall n \in N,n>2 \\
\end{array}
\]
__________________
$$\int_{0}^{\frac{\pi}{2}}\frac{a\cos x-b\sin x}{a\sin x+b\cos x}dx=\ln\left(\frac{a}{b}\right)$$
BUT
$$\int_{0}^{\frac{\pi}{2}}\frac{a\cos x+b\sin x}{a\sin x+b\cos x}dx=\frac{\pi ab}{a^{2}+b^{2}}+\frac{a^{2}-b^{2}}{a^{2}+b^{2}}\ln\left(\frac{a}{b}\right)$$

14 ธันวาคม 2006 23:09 : ข้อความนี้ถูกแก้ไขแล้ว 2 ครั้ง, ครั้งล่าสุดโดยคุณ Timestopper_STG
ตอบพร้อมอ้างอิงข้อความนี้
  #40  
Old 24 พฤศจิกายน 2006, 19:17
passer-by passer-by ไม่อยู่ในระบบ
ผู้พิทักษ์กฎทั่วไป
 
วันที่สมัครสมาชิก: 11 เมษายน 2005
ข้อความ: 1,442
passer-by is on a distinguished road
Smile

14. (Alternative Solution)

Let $ c_n = a_n + b_n $

So $ c_2= (a_1 + \frac{1}{a_1})+ (b_1 + \frac{1}{b_1}) $

It's easy to show that $ c_2 \geq 4 $ by AM-GM inequality.

Consider $ (c_{n+1})^2 $ with $ n \geq 2 $

$ \begin{array}{rcl }(c_{n+1})^2 &= & (a_n + b_n)^2 + 2(a_n + b_n)\big (\frac{1}{a_n}+\frac{1}{b_n}\big ) + \big (\frac{1}{a_n}+\frac{1}{b_n}\big )^2 \\ & \geq & (c_n)^2 +2(4)+ \big (\frac{1}{a_n}+\frac{1}{b_n}\big )^2 \\& > & (c_n)^2+ 8 \end{array}$

From above result, we can rewrite inequality in terms of $ c_2$ to be

$ (c_{2+k})^2 >(c_2)^2 +8k \geq 16+8k $

Clearly, if we select $ k \geq \frac{1000^2}{8}-2 = 124,998. $ It follows immediately that $ c_{2+k} > 1000 $
__________________
เกษียณตัวเอง ปลายมิถุนายน 2557 แต่จะกลับมาเป็นครั้งคราว
ตอบพร้อมอ้างอิงข้อความนี้
  #41  
Old 04 ธันวาคม 2006, 14:56
warut warut ไม่อยู่ในระบบ
กระบี่ไร้สภาพ
 
วันที่สมัครสมาชิก: 24 พฤศจิกายน 2001
ข้อความ: 1,627
warut is on a distinguished road
Smile

อ่า... เป็นโชคดีของผมที่คุณ passer-by มาเฉลยให้ดูซะก่อน ผมเลยไม่ต้องแสดงวิธีเฉิ่มๆออกไป จริงๆที่ผมทำตอนแรกก็เข้ามาทางที่คุณ passer-by แสดงให้ดูแล้วนะ แต่ผมดันไม่เอา $b_n$ มาร่วมคิดด้วย เพราะเดาว่ามันจะทำให้ยุ่งยากขึ้น แต่จากเฉลยจะเห็นว่าผมเดาผิด ทีนี้พอผมเอา $b_n$ กลับมาวิเคราะห์ต่อท้ายเข้าไป เลยกลายเป็นออกนอกลู่นอกทางไปซะงั้น

ส่วนข้อ 15. ของคุณ Timestopper_STG ผมประเมินว่าผมน่าจะทำได้นะครับ แต่ยังไม่รู้เหมือนกันว่าเมื่อไหร่จะมีเวลาได้ทด
ตอบพร้อมอ้างอิงข้อความนี้
  #42  
Old 07 ธันวาคม 2006, 23:15
passer-by passer-by ไม่อยู่ในระบบ
ผู้พิทักษ์กฎทั่วไป
 
วันที่สมัครสมาชิก: 11 เมษายน 2005
ข้อความ: 1,442
passer-by is on a distinguished road
Smile

ก่อนที่ผมจะสาบสูญจาก board ไป 1 เดือนเต็มๆ(จนถึงวันเด็ก) ขอแปะบางข้อทิ้งไว้ก่อน เดี๋ยวจะลืม

16. Simplify $$ \frac{{n \choose 0}}{1^{2}} - \frac{{n \choose 1}}{2^{2}} + \frac{{n \choose 2}}{3^{2}} - \cdots +(-1)^n \frac{{n \choose n}}{(n+1)^{2}} $$

17. (Cesaro mean) เราเคยพิสูจน์ไป(หลายครั้ง)แล้วว่า

ถ้า $ \lim_ {n \rightarrow \infty} a_n = a $ แล้ว $ \lim_ {n \rightarrow \infty} S_n = a $ เมื่อ $ S_n = \frac{a_1+a_2+\cdots +a_n}{n} $

และมีตัวอย่างมากมายที่แสดงว่า ถ้า $ a_n $ เป็น ลำดับที่มีขอบเขต (bounded sequence)
แล้วลิมิต ของ $ S_n $ ก็อาจหาค่าได้

ถามว่า จะมีลำดับไม่มีขอบเขต $ a_n $ อยู่บ้างหรือไม่ ที่ลิมิตของ $ S_n $ ก็ยังหาค่าได้


p.s. ไหนๆจะหายตัวไปชั่วคราว ก็ขอสวัสดีปีใหม่ล่วงหน้า Happy new year 2007 มีความสุข เฮง เฮง เฮงกันถ้วนหน้านะครับ
__________________
เกษียณตัวเอง ปลายมิถุนายน 2557 แต่จะกลับมาเป็นครั้งคราว
ตอบพร้อมอ้างอิงข้อความนี้
  #43  
Old 09 ธันวาคม 2006, 19:44
warut warut ไม่อยู่ในระบบ
กระบี่ไร้สภาพ
 
วันที่สมัครสมาชิก: 24 พฤศจิกายน 2001
ข้อความ: 1,627
warut is on a distinguished road
Smile

ไม่รู้คุณ passer-by มาแปะโจทย์ข้อ 16. เพื่อชี้แนะวิธีทำข้อ 65. ของคุณ nooonuii ใน Calculus Marathon หรือเปล่า แต่ยังไงผมว่ามันก็ช่วยให้ผมมองเห็นลู่ทาง การแก้ปัญหาของทั้งสองข้อแล้วล่ะ แต่ต้องขอลองมือทำจริงๆก่อนนะครับ ถึงจะมั่นใจว่าทำได้แน่

17. ให้ $$ a_n = \cases{\sqrt n & ,n \, \text{ is a perfect square} \\ 0 & , \text{ otherwise}}$$ จะเห็นว่า $\{a_n\}$ unbounded แต่เราได้ $$ \lim_{n \to \infty} \frac{a_1 + a_2 + \cdots + a_n}{n} = \frac12 $$ คิดว่าตัวอย่างนี้น่าจะใช้ได้นะครับ แต่ไม่รู้จะมีแบบอื่นที่ง่ายกว่านี้อีกหรือเปล่า

05 เมษายน 2007 15:18 : ข้อความนี้ถูกแก้ไขแล้ว 1 ครั้ง, ครั้งล่าสุดโดยคุณ gon
เหตุผล: Tag Post
ตอบพร้อมอ้างอิงข้อความนี้
  #44  
Old 10 ธันวาคม 2006, 17:42
warut warut ไม่อยู่ในระบบ
กระบี่ไร้สภาพ
 
วันที่สมัครสมาชิก: 24 พฤศจิกายน 2001
ข้อความ: 1,627
warut is on a distinguished road
Smile

อ้างอิง:
ข้อความเดิมของคุณ passer-by:
16. Simplify $$ \frac{{n \choose 0}}{1^{2}} - \frac{{n \choose 1}}{2^{2}} + \frac{{n \choose 2}}{3^{2}} - \cdots +(-1)^n \frac{{n \choose n}}{(n+1)^{2}} $$
โดยการ integrate $$ \sum_{k=0}^n {n \choose k} x^k = (1+x)^n $$ เราได้ $$ \sum_{k=0}^n {n \choose k} \frac{x^{k+1}}{k+1} = \frac{(1+x)^{n+1} -1}{n+1} $$ ดังนั้น $$ \sum_{k=0}^n {n \choose k} \frac{x^k}{k+1} = \frac{1}{n+1} \left( \frac{(1+x)^{n+1} -1}{(1+x)-1} \right) = \frac{1}{n+1} \sum_{k=0}^n (1+x)^k $$ integrate ตั้งแต่ -1 ถึง 0 เทียบกับ $x$ จะได้ $$ \sum_{k=0}^n {n \choose k} \frac{(-1)^k}{(k+1)^2} = \frac{1}{n+1} \sum_{k=0}^n \frac{1}{k+1} $$ นั่นคือ $$ \frac{{n \choose 0}}{1^{2}} - \frac{{n \choose 1}}{2^{2}} + \frac{{n \choose 2}}{3^{2}} - \cdots +(-1)^n \frac{{n \choose n}}{(n+1)^{2}} = \frac{1}{n+1} \left( 1+ \frac12 + \frac13 + \cdots + \frac{1}{n+1} \right) $$
ตอบพร้อมอ้างอิงข้อความนี้
  #45  
Old 12 ธันวาคม 2006, 02:45
warut warut ไม่อยู่ในระบบ
กระบี่ไร้สภาพ
 
วันที่สมัครสมาชิก: 24 พฤศจิกายน 2001
ข้อความ: 1,627
warut is on a distinguished road
Icon22

อ้างอิง:
ข้อความเดิมของคุณ Timestopper_STG:
\[
\begin{array}{l}
15)Let\;a_{n + 1} \;be\left\{ \begin{array}{l}
a_n + \frac{1}{n},a_n^2 < 2 \\
a_n - \frac{1}{n},a_n^2 > 2 \\
\end{array} \right.and\;also\;given\;a_1 = 1 \\
Show\;that\left| {a_n - \sqrt 2 } \right| < \frac{1}{n},\forall n \in N \\
\end{array}
\]
คุณ Timestopper_STG แต่งโจทย์ข้อ 15. นี้เองเหรอครับ ข้อความข้างบนไม่เป็นจริงนะครับ ยกตัวอย่างเช่น $ |a_2 -\sqrt 2| = 2-\sqrt2 \not< \frac12 $
ตอบพร้อมอ้างอิงข้อความนี้
ตั้งหัวข้อใหม่ Reply


หัวข้อคล้ายคลึงกัน
หัวข้อ ผู้ตั้งหัวข้อ ห้อง คำตอบ ข้อความล่าสุด
Alternating series (and Abel's theorem) Punk Calculus and Analysis 3 17 กรกฎาคม 2012 21:05
Marathon Mastermander ฟรีสไตล์ 6 02 มีนาคม 2011 23:19
On-Line Encyclopedia of Integer Sequences warut งานหรือข่าวคราวคณิตศาสตร์ทั่วไป 0 28 เมษายน 2007 00:28
ปัญหาชิงรางวัลข้อที่ 22: Infinite Series warut คณิตศาสตร์อุดมศึกษา 4 02 พฤศจิกายน 2006 05:35
Series intarapaiboon คณิตศาสตร์อุดมศึกษา 3 02 ตุลาคม 2005 10:58

เครื่องมือของหัวข้อ ค้นหาในหัวข้อนี้
ค้นหาในหัวข้อนี้:

ค้นหาขั้นสูง

กฎการส่งข้อความ
คุณ ไม่สามารถ ตั้งหัวข้อใหม่ได้
คุณ ไม่สามารถ ตอบหัวข้อได้
คุณ ไม่สามารถ แนบไฟล์และเอกสารได้
คุณ ไม่สามารถ แก้ไขข้อความของคุณเองได้

vB code is On
Smilies are On
[IMG] code is On
HTML code is Off
ทางลัดสู่ห้อง


เวลาที่แสดงทั้งหมด เป็นเวลาที่ประเทศไทย (GMT +7) ขณะนี้เป็นเวลา 04:15


Powered by vBulletin® Copyright ©2000 - 2024, Jelsoft Enterprises Ltd.
Modified by Jetsada Karnpracha